LSAT and Law School Admissions Forum

Get expert LSAT preparation and law school admissions advice from PowerScore Test Preparation.

 ceschulte
  • Posts: 1
  • Joined: Aug 19, 2016
|
#27922
I understand why the answer is A but why is E incorrect?
 Nikki Siclunov
PowerScore Staff
  • PowerScore Staff
  • Posts: 1362
  • Joined: Aug 02, 2011
|
#27930
Hi ceschulte,

Thanks for your question, and welcome to the Forum!

The stimulus is clear about the conditions under which restricting someone's liberty would be allowed: simply put, to prevent harm. Any harm. Meanwhile, answer choice (E) suggests that the government should not have restricted the publication of literature (a "liberty") that does not cause serious harm. But what if that literature still causes harm (even if it's not serious)? If it does, then maybe it's OK for the government to restrict it. Indeed, there might be cases in which basic harm would be sufficient justification for restriction of liberty. The stimulus does not specify the level of harm necessary for the government to take action.

Does this make sense? Let me know.

Thanks!
 jgray
  • Posts: 41
  • Joined: Feb 13, 2015
|
#32234
Could you re-diagram this problem from the beginning? Very confused on how to ID the s&n for liberty and offend tie into the first conditional statement.
 Kristina Moen
PowerScore Staff
  • PowerScore Staff
  • Posts: 230
  • Joined: Nov 17, 2016
|
#32261
jgray wrote:Could you re-diagram this problem from the beginning? Very confused on how to ID the s&n for liberty and offend tie into the first conditional statement.
Hi jgray,

"It is wrong for the government to restrict the liberty of individuals"

Restrict Liberty -> Wrong

"...except perhaps in those cases when to fail to do so would allow individuals to cause harm."

This isn't really a conditional statement per se, as it says "perhaps." However, what we know is that, if the government restricts a liberty when failing to restrict it would allow individuals to cause harm, then it might not be wrong. (e.g. restricting my freedom to assault someone might not be wrong).

"Yet, to publish something is a liberty, and to offend is not to cause harm."
Publish -> Liberty and Offend -> No harm.

So we can infer that it would be wrong to restrict publications (a liberty) even if it were offensive (because it wouldn't cause harm).

Hope that helps.
 mankariousc
  • Posts: 32
  • Joined: Feb 13, 2017
|
#33093
Hello!
For some reason, I can't find the original post for this question with each answer choice explained. Could you explain why the answer choice is not D and why it is A?
Also, I was thrown off by the "it is not right" part of choice A. When I was prephrasing I was looking for something that said "it is not wrong." Could you explain why that was wrong?
User avatar
 Jonathan Evans
PowerScore Staff
  • PowerScore Staff
  • Posts: 726
  • Joined: Jun 09, 2016
|
#33131
Hi, Mankariousc,

Good question. Let's address this question step-by-step.

First, we should observe that the stimulus is a fact set. There is no explicit conclusion (you could make the case that there is a tacit conclusion here, but that's beside the point).

In your analysis, you should take note of the facts at your disposal and perhaps observe any salient connections you can make among different facts.

You should proceed with as much mental precision as possible, such that your analysis might be something to the effect of:

(1) harm :arrow: gov't wrong to restrict individual liberty

(2) publication :arrow: liberty

(3) offend :arrow: harm

With this analysis, you are equipped to consider the validity of the answer choices. (I'll mention an additional possible prephrase in a moment).

To address your second point about your prephrase about expecting to see "it is not wrong" while actually presented with "it is not right," this is actually a skill you should endeavor to develop, that is you should be able to consider what each of these statements means rephrased or expressed differently.

For instance, let's look at each of these phrases:

"it is not wrong" :dbl: (morally neutral OR "it is right")

AND

"it is not right" :dbl: (morally neutral OR "it is wrong")

Sometimes it is necessary to consider these phrases from a slightly different angle to find the credited response. Now back to our analysis of the stimulus.

Answer choice (A): Let's take this one step at a time. What do we know about the publication of literature? It is a liberty. What do we know about liberties? It is wrong for the government to restrict them unless perhaps they cause harm. Therefore, given that publication does not cause harm, it is categorically wrong to restrict such a liberty. Now, connect the other information. What do we know about something that is only offensive? It does not cause harm. Therefore we know that this particular publication is a liberty that does not cause harm. Thus, it would be wrong to restrict such publication.

Return to our explanation of what it would mean for something to be wrong:

"it is not right" :dbl: (morally neutral OR "it is wrong")

Thus, to be wrong is in fact a sufficient condition to know something "is not right"

"it is wrong" :arrow: "it is not right"

Thus, (A) is supported.

Consider answer choice (D) now. Notice something about this answer choice; we are no longer talking about what is right or wrong for the government to do but rather what is right or wrong for an individual to do. The stimulus fails to address whether an individual publishing something offensive is right or wrong, only whether a government is right or wrong to restrict such publication.

Here's an analogy.

You and I might agree it would be inappropriate for the government to forbid my daughter to take away my son's ice cream sandwich; however, I would argue that my daughter might get in trouble for swiping my son's ice cream sandwich, because this is not very nice, and I consider such behavior wrong. :)

Let's take another look at our initial analysis to see what else we might have been able to do to prephrase. To recap:

(1) harm :arrow: gov't wrong to restrict individual liberty

(2) publication :arrow: liberty

(3) offend :arrow: harm

If you've tracked all this stuff mentally, you might ask yourself, "so where's this guy going?" In this case, I believe there is a bit of an implicit conclusion, that in fact it's wrong for the government to restrict publication of literature that is only offensive. We've seen how answer choice (A) is consistent with this analysis.

But let's leave that aside and see what we can connect. We could take (1) and (3) to conclude that offense by itself implies it is incorrect for the government to restrict such liberty.

offend :arrow: harm :arrow: gov't wrong to restrict individual liberty

Now you could also infer that since "publication" is necessarily a liberty, it would be wrong for the gov't to restrict this publication were it only to offend.

The only remaining step is to understand how "it is wrong" is logically consistent with saying "it is not right." At a minimum something that is wrong is certainly not right.

I know this was a long discussion, but let's break it down into a couple takeaways:
  1. Endeavor to achieve a clear analysis of the stimuli so that you have distilled them down to the basic facts.
  2. If you can anticipate connections in a Must Be True stimulus, great, if not, don't sweat it. Just be clear on what it is that you do know.
  3. Understand that the LSAT can change the way things are phrased as long as meaning is consistent, as happens here. Expect this to happen.
 mankariousc
  • Posts: 32
  • Joined: Feb 13, 2017
|
#33142
This was so helpful! Thank you so much!
 co659
  • Posts: 10
  • Joined: Apr 23, 2017
|
#34306
Hello ! I've read through all the explanations here and I'm still really confused how A is the correct answer. When I first looked at this I crossed everything off so perhaps there's an issue with my diagramm:

RL = restrict liberties
CH = cause harm
P = publishing
O = to cause offense
L = liberty

STIMULUS
1) RL :arrow: CH :arrow: /O
CP: O :arrow: C/H :arrow: R/L

2) P :arrow: L

ANSWER CHOICES:
A) I viewed "only" as a necessary indicator so I diagrammed it as such:
R/L :arrow: O

Based off the chain conditional phrases, (A) would be incorrect, right ?

Thanks,
 Kristina Moen
PowerScore Staff
  • PowerScore Staff
  • Posts: 230
  • Joined: Nov 17, 2016
|
#34351
Hi co659,

Although I diagrammed this stimulus further up on the thread, I find it easier to approach this question without conditional diagrams.

"It is wrong for the government to restrict the liberty of individuals EXCEPT perhaps in those cases when to fail to do so would allow individuals to cause harm." This is a principle, and there's ONE exception which involves harm. Right now, there's nothing to apply the principle towards. You might also think of this as a rule with one exception.

"Yet, to publish something is a liberty..." Now we have a specific liberty. Publishing! We can apply the rule! Can we restrict it or not? Only if it falls under the ONE exception, which is if it causes harm.

"And to offend is not to cause harm...." Okay, so offending someone is not a harm. So if I publish something that offends my neighbor, it's not causing harm. So that wouldn't fall under the exception.

What you have here is a pretty standard Must Be True. We're given a general rule/principle and a specific situation. The answer choice just applies the rule to the situation.

Answer choice (A) uses the word "only," so it is tempting to diagram it as a conditional. However, what it's really saying is that "It is not right for the government to restrict the publication of literature that is only offensive does not cause harm." It's just like if you said "I only had a bologna sandwich for lunch." That means you did not have a PBJ or a ham sandwich or any other sandwich. Thus, answer choice (A) is talking about publication (liberty!) that does not cause harm (not an exception to the rule!). And if we apply the rule, then it would be wrong to restrict that liberty.

Hope this helps.
 mariahenain
  • Posts: 30
  • Joined: Jun 09, 2017
|
#36164
I'm still confused on the choice A being correct. For some reason, the wording and negation of the stimulus is making it difficult to diagram (and difficult to understand the diagram you drew). Could you explain the process of diagramming this question?

Thank you!
  • 1
  • 4
  • 5
  • 6
  • 7
  • 8

Get the most out of your LSAT Prep Plus subscription.

Analyze and track your performance with our Testing and Analytics Package.